5
$\begingroup$

Suppose that a continuous function $f$ on the line and satisfies $$ |f(x+2h)−2f(x+h)+f(x)|\leq const \frac{|h|}{(\log\frac{1}{|h|})^{\beta}}\,\,\,\,\,\,\text{where}\,\,\,\, \beta \in(0, 1] $$ for all $x,h$ real. Is it true that $f$ is differentiable? If not how can I prove it?

$\endgroup$
1
  • $\begingroup$ The Article "A Note on Smooth Functions" from Weiss and Zygmund is information about this class of functions at least in the periodic case. Maybe something can be generalized. goo.gl/ha7i4l $\endgroup$
    – CPJ
    Sep 19, 2014 at 15:57

1 Answer 1

12
$\begingroup$

The Weierstrass-type function

$$ f(x) := \sum_{n=1}^\infty \frac{1}{2^n n^\beta} \sin(2^n x) $$

will obey the hypotheses, yet fails to be differentiable at the origin.

In general, one should look to Weierstrass-type functions (perhaps weighted by power weights $|x|^{-\alpha}$ or variants such as $|x|^{-\alpha} \log(\frac{1}{|x|})^{-\gamma}$, in case some $L^p$ norms are involved for a finite $p$) as key test cases for these sorts of endpoint functional embedding problems. It is also quite clarifying to reformulate the hypotheses and conclusion in terms of Littlewood-Paley theory (either the classical theory using harmonic extensions, or the modern theory using smooth partitions of unity in frequency space; see e.g. Stein's "Singular integrals" for the former, or the appendix to my PDE book for the latter). For instance, the hypothesis here is basically equivalent to the Besov-type bound

$$ \| P_N f \|_\infty \ll N^{-1} \log^{-\beta} N$$

for frequencies $N \gg 1$, where $P_N$ is a smooth Fourier projection to frequencies $|\xi| \sim N$, while the conclusion is roughly equivalent to the pointwise convergence of the series $\sum_N N P_N f(x)$ as $N$ ranges over dyadic integers. As $\sum_N \log^{-\beta} N$ diverges for $\beta \leq 1$, this indicates that the claim is false, and then guided by this analysis one can quickly come up with the aforementioned Weierstrass-type counterexample.

$\endgroup$
2
  • $\begingroup$ Thank you very much Terry Tao. If we suppose $f$ is absolutely continuous and satisfies above inequality, in this case, is it true that $f$ is differentiable and its derivative satisfies $$ |f'(x)-f'(y)|\leq\frac{1}{(\log\frac{1}{|h|})^{\beta}} $$Thanks $\endgroup$
    – Ravi
    Oct 2, 2013 at 5:36
  • $\begingroup$ No, and one can see this by applying appropriate truncation to each of the terms in the previous example (note that absolute continuity is equivalent to the weak derivative of f being in L^1). It would probably be a good exercise for you to work out the details. $\endgroup$
    – Terry Tao
    Oct 2, 2013 at 20:08

Your Answer

By clicking “Post Your Answer”, you agree to our terms of service and acknowledge you have read our privacy policy.

Not the answer you're looking for? Browse other questions tagged or ask your own question.